Anda di halaman 1dari 54

INTENSIVE TOEFL TRAINING

PREPARATION GUIDE

Untuk kalangan sendiri

Penanggung Jawab
Mohammad Noor Zuhri, S.Pd., M.Pd.

Pengarah
Kurnia Citra Dewi, S.Pd., M.Pd.

Tim Penyusun
Anisykur Dzakiyah, S.Pd., M.Pd.
Nawang Wulan, S.Pd., M.Pd.
Asa Bani Putria, S.Pd.

Penyunting
Nawang Wulan, S.Pd., M.Pd.

Tata Letak dan Desain Sampul


Dimas Mahardika Ilmi

Penerbit
UPT Pengembangan Bahasa Internasional
(CILAD - Center for International Language Development)

Terbit Pertama 2018


Terbit Kedua 2019

Alamat Redaksi
UPT Pengembangan Bahasa Internasional (CILAD)
Universitas Islam Sultan Agung Semarang
Jl. Raya Kaligawe Km. 4 Semarang, Jawa Tengah, 50112
Telepon (024)6583584 ext. 207
Surel: cilad@unissula.ac.id
Laman: www.cilad.unissula.ac.id
KATA PENGANTAR

Assalamualaikum Wa Rahmatullah Wa Barakatuh


Alhamdulillah segala puji bagi Allah sang pencipta alam semesta. Kepada-Nya lah
kita memohon ampunan dan Ridho-Nya. Sholawat serta salam senantiasa tercurahkan kepada
baginda Nabi Muhammad Shallahu „Alaihi Wa Sallam yang telah menuntun kita dari jalan
kegelapan menuju jalan kebenaran. Semoga kelak kita akan mendapatkan pertolongan beliau
di hari kiamat. Amin.
Syukur kami panjatkan atas kenikmatan berupa terwujudnya modul Tutorial TOEFL
UPT Pengembangan Bahasa Internasional. Dengan berbagai macam quality control dari tim
akademik UPT PBI serta proses validasi oleh tim ahli, akhirnya modul ini dapat diterbitkan.
Modul ini merupakan sebuah ikhtiar kami untuk memberikan latihan-latihan dan kisi-kisi
terkait dengan Ujian TOEFL-Like di Lingkungan Universitas Islam Sultan Agung Semarang.
Oleh karena itu, kami sangat berharap modul ini dapat memberikan manfaat dan
kemudahan bagi para mahasiswa di Universitas Islam Sultan Agung Semarang untuk dapat
meraih nilai minimal kelulusan sebagai syarat kelulusan.
Wa billahit taufiq wal hidayah.
Wassalamualaikum Wa Rahmatullah Wa Barakatuh.

Kepala UPT Pengembangan Bahasa Internasional

Mohammad Noor Zuhri, S.Pd., M.Pd


NIK. 110014520
DAFTAR ISI

PENDAHULUAN .......................................................................................................................... 3
TOEFL ................................................................................................................................... 3
STRUKTUR TOEFL ............................................................................................................. 3
KLASIFIKASI UMUM NILAI TOEFL................................................................................ 3
PENILAIAN TOEFL............................................................................................................. 3

LISTENING COMPREHENSION .............................................................................................. 4


1.1 DIALOG PENDEK .................................................................................................................. 4
Skill 1: Focus on the Last Line and Restate with Synonym ..................................................... 4
TOEFL Exercise 1.................................................................................................................... 5
Skill 2: Who, What, and Where .............................................................................................. 5
TOEFL Exercise 2.................................................................................................................... 6
Skill 3: Agreement ................................................................................................................... 6
TOEFL Exercise 3.................................................................................................................... 7
Skill 4: Avoid Similar Sounds .................................................................................................. 7
TOEFL Exercise 4.................................................................................................................... 8
1.2 LONG CONVERSATION ....................................................................................................... 8
TOEFL Exercise 5.................................................................................................................... 10
TOEFL Exercise 6.................................................................................................................... 10
1.3 LONG TALKS ......................................................................................................................... 10
TOEFL Exercise 7.................................................................................................................... 12
TOEFL Exercise 8.................................................................................................................... 12
REVIEW LISTENING COMPREHENSION............................................................................. 13

STRUCTURE & WRITTEN EXPRESSION .............................................................................. 16


Skill 1: Be Sure the Sentence Has a Subject and a Verb ................................................................. 16
Exercise 1 ......................................................................................................................................... 16
Exercise 2 ......................................................................................................................................... 17
Skill 2: Be Careful of Objects of Prepositions ................................................................................. 17
Exercise 3 ......................................................................................................................................... 18
Exercise 4 ......................................................................................................................................... 18
Skill 3: Make Verbs Agree after Certain Words .............................................................................. 19
Exercise 5 ......................................................................................................................................... 19
Exercise 6 ......................................................................................................................................... 20
Skill 4: After Have, Use the Past Participle ..................................................................................... 20
Exercise 7 ......................................................................................................................................... 21
Exercise 8 ......................................................................................................................................... 21
Skill 5: After Will, Would, or Other Modals, Use the Base Form of the Verb................................ 22
Exercise 9 ......................................................................................................................................... 22
Exercise 10 ....................................................................................................................................... 23
REVIEW STRUCTURE & WRITTEN EXPRESSION ............................................................ 24

READING COMPREHENSION.................................................................................................. 28
Skill 1: Answer Main Idea Questions Correctly .............................................................................. 28
Exercise 1 ......................................................................................................................................... 29
Skill 2: Answer Stated Detail Questions Correctly .......................................................................... 30
Exercise 2 ......................................................................................................................................... 31
Skill 3: Find Pronoun Referents ....................................................................................................... 31
Exercise 3 ......................................................................................................................................... 32
READING EXERCISES ............................................................................................................... 33
REVIEW READING COMPREHENSION ................................................................................ 35

Center for International Language Development | 1


TRY-OUT 1 .................................................................................................................................... 40
TRY-OUT 2 .................................................................................................................................... 46

Center for International Language Development | 2


PENDAHULUAN
Apa itu TOEFL?

TOEFL merupakan singkatan dari Test of English as a Foreign Language. TOEFL adalah
standardisasi kemampuan bahasa Inggris seseorang secara tertulis yang meliputi empat aspek
penguasaan: Listening, Writing, Reading, dan Speaking. Namun, untuk model Paper Based Test
hanya terdapat tiga aspek yang diujikan: Listening Comprehension, Structure and Written Expression,
dan Reading Comprehension
Disamping itu, ada kalanya TOEFL dipakai dalam bidang umum seperti pekerjaan, kenaikan
pangkat atau tugas kerja. Banyak perusahaan yang memasang standar Bahasa Inggris karyawannya
dengan melihat nilai TOEFL. Umumnya, nilai TOEFL minimal adalah 450-500 untuk kenaikan
pangkat standar.

Struktur TOEFL

Ada tiga bagian tes TOEFL yang harus dikerjakan oleh peserta. Bagian pertama adalah soal- soal
yang mengukur kemampuan Listening Comprehension (50 soal), Structure & Written Expression (40
soal), dan Reading Comprehension (50 soal). Keseluruhan soal dibuat dalam bentuk pilihan berganda.
Keseluruhan tes berlangsung kurang lebih 120 menit, untuk Paper Based Test (PBT), dan kurang
lebih 240 menit untuk Internet Based Test (IBT).

Klasifikasi UMUM Nilai TOEFL

Secara umum kita mengenal tiga level penguasaan bahasa asing, yaitu Tingkat Dasar (Elementary),
Tingkat Menengah (Intermediate), dan Tingkat Mahir (Advanced). Untuk skor TOEFL, para ahli
bahasa biasanya mengelompokkan skor ini kedalam empat level berikut (Carson, et al., 1990):

 Tingkat Dasar (Elementary) : 310 s.d. 420


 Tingkat Menengah Bawah (Low Intermediate) : 420 s.d. 480
 Tingkat Menengah Atas (High Intermediate) : 480 s.d. 520
 Tingkat Mahir (Advanced) : 525 s.d 677

Penilaian TOEFL

Sistem penilaian TOEFL menggunakan konversi dari setiap jawaban yang benar. Nilai TOEFL
tertinggi yang bisa dicapai seseorang adalah 677.

Center for International Language Development | 3


LISTENING COMPREHENSION
Waktu = 35 menit (termasuk pembacaan petunjuk pelaksanaan
untuk setiap bagian) Tes ini memberikan kesempatan untuk
menunjukkan kemampuan memahami percakapan dan
pembicaraan dalam bahasa Inggris. Listening Comprehension Test
terdiri dari 3 (tiga) bagian:

Part A : SHORT DIALOGUE (DIALOG PENDEK)


Pada bagian ini, terdapat percakapan singkat antara dua orang.
Setelah percakapan, pertanyaan yang berhubungan dengan
percakapan tersebut akan diberikan. Baik percakapan dan
pertanyaan tidak akan diulangi.

Part B : LONG CONVERSATION (PERCAKAPAN PANJANG)


Pada bagian ini, terdapat percakapan dengan topik santai antara beberapa orang dengan durasi sekitar
60-90 detik. Setelah percakapan panjang selesai akan diikuti oleh sejumlah pertanyaan pilihan ganda
yang kesemuanya merujuk pada percakapan panjang tersebut.

Part C : TALKS (PEMBICARAAN/CERAMAH)

1
Pada bagian ini, terdapat beberapa pembicaraan dengan durasi sekitar 60-90 detik tentang kehidupan
sekolah atau perkuliahan, masing-masing diikuti dengan pertanyaan pilihan ganda.

1.1 DIALOG PENDEK


SKILL 1: Focus on the Last Line and Restate with Synonym
Seringkali jawaban yang benar untuk sebuah pertanyaan dalam bagian dialog pendek adalah
jawaban yang berisi penyajian kembali ide-ide dalam baris terakhir dari dialog. Jika anda tidak
sempat menangkap maksud speaker pertama, maka jangan khawatir karena biasanya informasi
terkait jawaban berada pada speaker kedua. Kemudian, cobalah untuk menyatakannya kembali ke
dalam sebuah kalimat lain yang memiliki kesamaan arti.

Example :
On the recording, you hear:
(Jane) Steve, is something the matter? You don‟t look very good.
(Steve) Oh, I‟m feeling a little sick today.
(Narrator) What does the man mean?

In your test book you read:


(A) He‟s a bit ill.
(B) He‟s very good looking.
(C) He looks worse than he feels.
(D) His feet are a little thick.

Center for International Language Development | 4


Garis Bawahi Ide Pokok dari Dialog Singkat di Bawah Ini.

(Jannah) What do you like about your new (Herry) Mark said some really nice things to me.
house? (Jannah) He‟s very grateful for what you did
(Herry) It‟s very close to a park (Narrator) What does the woman say about
(Narrator) What does the man mean? Mark?

(A) His home is near a park. (A) He‟s thankful.


(B) The house is closed up now. (B) He made a great fool of himself.
(C) He parks his car close to his house. (C) He bought a crate full of fruit.
(D) He doesn‟t really like his new house. (D) He did a great job.

TOEFL EXERCISE 1
In this exercise, listen carefully to each short dialogue and question on the recording, and then choose
the best answer to the question. You should look for synonyms for key words in the last line.

Now begin the recording at TOEFL exercise 1


1. (A) There were two exams rather than one. 3. (A) It was just established that he could go
(B) The final exam was harder than the into business.
others. (B) The company was founded about a
(C) He thought the exam would be easier. year ago.
(D) The exam was not very difficult. (C) The business only lasted a year.
(D) The family is well-established.
2. (A) He‟d really rather not answer.
(B) He‟s not feeling very well. 4. (A) He had to wait for the plane to land.
(C) He‟s feeling better today than (B) The plane arrived on time.
yesterday. (C) The plane landed in the right place.
(D) He‟s rather sick of working the (D) He did not look at the right schedule.
question.
5. (A) She needs a sweat suit to go running.
(B) She‟ll change clothes quickly and go

2
swimming
(C) She doesn‟t want to go into the pool.
(D) She‟s rather go running.

Skill 2: WHO, WHAT, and WHERE (Siapa, Apa, dan Dimana)

Keduanya sering digunakan dalam dialog singkat dengan tujuan menarik kesimpulan. Dalam hal
ini jawabannya tidak dinyatakan dengan jelas, melainkan harus dapat disimpulkan berdasarkan
klausa yang diberikan dalam dialog. Salah satu jenis kesimpulan yang umum di bagian tes ini
adalah meminta Anda untuk menentukan siapa pembicara itu, apa aktifitas yang dilakukan, dan
dimana tempat percakapan tersebut dilakukan berdasarkan klausa dalam dialog.

Example I :
On the recording, you hear:
(Yusuf) What do you do during your performance?
(Maria) I play the piano and sing
(Narrator) Who is the woman most likely to be?

In your test book, you hear:


(A) An athlete Petunjuk piano dan bernyanyi dalam
(B) A member of the audience percakapan menggiring kita pada kesimpulan
(C) Musician bahwa wanita itu seorang musisi sehingga (C)
(D) A clerk in a music store adalah jawaban yang benar.

Center for International Language Development | 5


Example II:
On the recording, you hear: In your test book, you read:
(Hendry) I‟d like to open an account, please (A) In an accounting class
(Joana) What type of account would you like, a (B) In a bank
checking or savings account? (C) At a service station
(Narrator) Where does the conversation (D) In a market
probably take place?

Dalam latihan ini, baca dialog singkat dan pertanyaan dibawah, garisbawahi petunjuk yang bisa
membantu dan pilih jawaban terbaik.

1. (Hendry) I‟d like to mail this package, 2. (Maria) How much are the flowers?
please. (Malik) Ten dollars a bouquet
(Joana) First or second class? (Narrator) Where does this
(Narrator) Who is the woman most likely to conversation probably take place?
be?
(A) A postal worker (A) In a florist shop
(B) A school administrator (B) In a garden
(C) A teacher (C) In a bakery
(D) A banker (D) In a grocery store

TOEFL EXERCISE 2
In this exercise, listen carefully to each short dialogue and question on the recording and then choose
the best answer to the question. You will have to draw conclusions about who, what, and where

1. (A) In a biology laboratory (C) Wait until later to clean up.


(B) In a photography studio (D) Wash the dishes for as long as possible.
(C) In the library
4. (A) In restaurant
(D) In an office
(B) In a bank
2. (A) He‟s flight attendant. (C) In a beauty salon
(B) He‟s a pilot (D) At a service station
(C) He works clearing land.
5. (A) A shoe repair person
(D) He‟s a member of the ground crew
(B) A salesclerk in a shoe store

3
3. (A) Use as many dishes as possible. (C) A salesclerk in a fixtures department
(B) Wash the dishes immediately. (D) A party caterer

Skill 3: AGREEMENT (Persetujuan)

Ungkapan persetujuan lazim digunakan dalam dialog singkat. Beberapa kata kunci yang
mengindikasikan persetujuan: Me, too, I‟ll say, you can say that again, I do either, neither am I.
Berikut contoh yang menunjukkan perjanjian dengan pernyataan positif:

Example III:
On the recording, you hear: In your test book, you read:
(John) I thought that the meal was 1. She has the same opinion of the meal as the
overpriced. man.
(Shirley) Me, too. 2. There were too many spices in the meal.
(Narrator) What does the woman mean? 3. The price of the meal was great.
4. She wants to share the man‟s meal.
Ungkapan “Me, too” menunjukkan persetujuan dalam hal positif. Hal ini berarti si wanita setuju
dengan
5. pendapat si laki-laki. Sehingga (A) adalah jawaban yang paling tepat.

Center for International Language Development | 6


Garis bawahi ekspresi yang menunjukkan persetujuan di setiap dialog singkat. Pilih
jawaban terbaik untuk pertanyaan itu. Ingat bahwa jawaban terbaik adalah yang
menunjukkan persetujuan

1. (Ani) This homework is going to take 2. (Windy) A trip to the park might be nice?
forever. (Bukhori) You can say that again.
(Ahmad) I‟ll say. (Narrator) What does the man mean?
(Narrator) What does the man mean? (A) It‟s nice in the park at night.
(A) It‟s going to take forever to get home. (B) The woman should repeat what she said.
(B) It takes a long time to get from home to (C) He agrees about the trip to the park.
work. (D) The woman should tell him about part of
(C) He and the woman have the same the trip.
opinion about the homework.
(D) He needs to take the homework to class.

TOEFL EXERCISE 3
In this exercise, listen carefully to each short dialogue and question on the recording and then choose
the best answer to the question. Remember that the best answer is one that shows agreement.

1. (A) She would also like to take a trip. (C) She is not sure which course she
(B) She would like to take two trips should take.
rather than one. (D) She‟s not sure if she should take a
(C) She doesn‟t think that a trip would trip to France.
be a good idea.
4. (A) The man said something foolish.
(D) The trip would cost too much.
(B) The man should repeat what he said.
2. (A) He thinks the elections should take (C) She thinks that the food is the best
place next month. she has ever tasted.
(B) He disagrees with the woman. (D) She agrees that the food is pretty bad.
(C) He would like to see the elections for
town council. 5. (A) He wants to know what he said.
(D) He agrees that Matt should be (B) He wonders if the woman enjoyed
herself.
elected.
(C) This party hasn‟t been any fun at all.
3. (A) Knows that she is not ready for (D) He‟s enjoyed himself tremendously.

4
intermediate French.
(B) She wants to take neither beginning
nor intermediate French.

Skill 4: AVOID SIMILAR SOUNDS (Hindari Suara yang Sama atau Mirip)
Seringkali jawaban yang salah dalam percakapan pendek adalah jawaban yang mempunyai suara
sama atau mirip namun memiliki perbedaan arti yang sangat jauh dengan apa yang didengar dalam
percakapan. Oleh Karena itu, hindarilah jawaban-jawaban tersebut.

Example III:
On the recording, you hear:
(David) Why couldn‟t Mark come with us? Di dalam percakapan tersebut terdapat kata
(Shirley) He was searching for a new apartment. searching dan apartment dimana bunyi
(Narrator) What does the woman say about Mark? kata-kata tersebut mirip dengan kata
In your test book, you read: department, research, dan church. Oleh
(A) He was working on his research project. Karena itu, jawaban yang paling tepat
(B) He had an appointment at church. adalah D karena tidak memiliki kata yang
(C) He was in the department office. sama atau mirip.
(D) He was looking for a place to live

Center for International Language Development | 7


Garis bawahi ekspresi yang menunjukkan persetujuan di setiap dialog singkat.
Pilih jawaban terbaik untuk pertanyaan itu. Ingat bahwa jawaban terbaik adalah suara
yang tidak sama atau tidak mirip dengan dialog

1. (Devi) I heard that Sally just moved into a 2. (Santi) Did they get the new car they
new, big house near the beach. wanted?
(Danu) But Sally doesn‟t have a cent! (Sulaiman) No, they lacked the money.
(Narrator) What does the man mean? (Narrator) What does the man mean?
(A) Sally has no money. (A) They locked the map in a car.
(B) Sally is on the set with her. (B) They looked many times in the car.
(C) Sally has no sense of responsibility. (C) It cost a lot of money when the car
(D) Sally sent her friend to the house. leaked oil.
(D) They didn‟t have enough money to
buy another car.

TOEFL EXERCISE 4
In this exercise, listen carefully to each short dialogue and question on the recording and then choose
the best answer to the question. Remember that you should avoid similar sound.

1. (A) The lawn is too dry. (C) He has been traveling back and forth
(B) She needs to watch out for a crash. to Boston.
(C) She has to wait for some cash. (D) He has been regularly using computer
(D) The waiter is bringing a glass of
4. (A) He learned a massive number of details.
water.
(B) He didn‟t like most of the lesson.
2. (A) The sweater‟s the wrong size (C) He thought the lesson didn‟t matter.
(B) The man‟s feet aren‟t sweating (D) He couldn‟t learn the lesson
(C) The sweet girl doesn‟t feel right
5. (A) In the first frost, animals die
(D) The sweater makes the man seem fat
(B) Frost can kill animals
3. (A) He regularly goes to communities (C) Some animals started the first fire
around Boston. (D) Animals are killed by forest fires
(B) He communicates with a Boston
company.

1.2 LONG CONVERSATION (Percakapan Panjang)


Pembicaraan panjang seringkali berkisar tentang beberapa persoalan terkait studi (betapa sulitnya
sebuah mata kuliah, bagaimana menulis sebuah makalah penelitian, bagaimana cara mendaftar
untuk kursus, dsb) atau tentang kehidupan secara umum (menyewa apartemen, bermain olahraga,
pergi ke bank). Percakapan juga dapat berupa kehidupan umum di Amerika Serikat (desalinasi
pasokan air, daur ulang produk yang digunakan, kerusakan dari badai atau tipe lain dari
fenomena kapan panjang akan diikuti beberapa buah pertanyaan yang merujuk dari tema yang
sama.

Center for International Language Development | 8


Example III:
(Narrator) Question 1 through 4. Listen to a conversation between two people who are
decorating an apartment.
(Woman) Hey, Walt. Do you think you could help me hang these pictures on the wall? There are
only two of them.
(Man) Sure, Monica. Where do you want them to go?
(Woman) I‟d like to picture of the mountains over the fireplace, and I‟d like the picture of my
family over the sofa. What do you think?
(Man) I think they‟ll look fine there. How about if you hold the pictures while I hammer the
nails into the wall?
(Woman) Okay. Let‟s start with the picture of my family.

Questions:

On the recording, you hear:


(Narrator) What are the man and woman discussing?

In your test book, you read: Karena si wanita meminta si pria itu untuk
(A) Hanging some plants membantu menggantung gambar-gambar di dinding,
(B) Taking some pictures jawaban terbaik untuk pertanyaan ini adalah
(C) Putting some pictures on the wall jawaban ini (C), menarik beberapa gambar di
(D) Taking a trip to the mountains dinding.

On the recording, you hear:


(Narrator) How many pictures are there?

In your test book, you read: Pertanyaan kedua bertanya berapa banyak gambar
(A) Three yang ada, dan si wanita dengan jelas mengatakan
(B) Four bahwa ada dua, jadi jawaban terbaik adalah
(C) One jawaban (D).
(D) Two

On the recording, you hear:


(Narrator) Where is the picture of the woman‟s family going?

In your test book, you read: Pertanyaan ketiga bertanya tentang lokasi gambar
(A) To the top of the mountain keluarga. Wanita itu mengatakan bahwa dia akan
(B) Home with Walt meletakkannya di atas sofa, jadi jawaban terbaik
(C) Above the sofa untuk pertanyaan ini adalah jawaban (C), di atas
(D) in the fireplace sofa.

On the recording, you hear:


(Narrator) What is Walt probably going to do next?

In your test book, you read: Pertanyaan terakhir menanyakan apa yang mungkin
(A) Photograph Monica‟s family akan dilakukan oleh Walt. Walt telah menyarankan
(B) Sit on the sofa bahwa ia harus memasang paku-paku ke dinding,
(C) Climb the walls jadi jawaban terbaik adalah jawaban (D).
(D) Hammer the nails into the walls

Center for International Language Development | 9


TOEFL EXERCISE 5

Listen to each complete conversation and answer the questions that follow.

1. (A) Attend a football game alone 4. (A) A play


(B) Go to a sporting event (B) A game
(C) Eat in the cafeteria and study (C) A study group meeting
(D) See a play (D) Dinner in the cafetaria
2. (A) It‟s the final game of the season 5. (A) Saturday night
(B) It‟s better than the drama department‟s (B) After dinner in the cafetaria
play (C) Sunday afternoon
(C) It‟s a very important game (D) Maybe next weekend
(D) It‟s close to the cafetaria
3. (A) At the cafetaria
(B) At the music department
(C) At the campus
(D) At the drama department

TOEFL EXERCISE 6
Listen to each complete conversation and answer the questions that follow.

1. (A) The haircut is unusually short. 3. (A) To go to town


(B) This is Bob's first haircut. (B) To cut all of his hair
(C) Bob doesn't know who gave him (C) To trim his hair just a little bit
the haircut. (D) Put his hair on the floor
(D) After the haircut, Bob's hair still 4. (A) A broken mirror
touches the floor.
(B) The hairstylist
2. (A)It is just what he wanted. (C) The scissors used to cut his hair
(B) He enjoys having the latest style. (D) Piles of his hair
(C) He dislikes it immensely.
5. (A) "You should become a hairstylist."
(D) He thinks it will be cool in the
summer (B) "Please put it back on."
(C) "It'll grow back."
(D) "It won't grow fast enough."

1.3 LONG TALKS (Pembicaraan Panjang)


Topik pembicaraan sering tentang beberapa aspek kehidupan sekolah atau topik dalam berita.
Pembicaraan panjang bisa juga diperpendek dengan tema perkuliahan di perguruan tinggi
Amerika dan universitas luar negeri lainnya. Dari sebuah long talks akan diberikan beberapa
pertanyaan terkait pembicaraan tersebut.

Center for International Language Development | 10


Example III:
On the recording, you hear:
(Narrator) Question 1 through 4. Listen to a lecture in a history class
(Woman) Salt, which today seems so plentiful to us, in the past used to be a valuable
commodity. In the ancient past in China, salt was used to make coins, and in parts of Africa
it was traded in place of money. In the Roman Empire, soldiers were not paid in money but
were instead paid in salt. In fact, the English word “salary” which means the amount of
money that one earns, comes from the latin root for “salt”.

Questions:

On the recording, you hear:


(Narrator) What is the topic of the talk? Pertanyaan pertama menanyakan tentang topik
In your test book, you read: pembicaraan. Pembicaraan dimulaidengan topik
(A) Ancient China garam dan terus berbicara tentang hal itu di
(B) Money seluruh bagian, jadi jawaban terbaik adalah
(C) Valuable commodities jawaban (D).
(D) Salt

On the recording, you hear:


(Narrator) What was salt used for in China? Pertanyaan kedua menanyakan tentang
In your test book, you read: penggunaan garam di Cina. Pembicara
(A) To locate Africa mengatakan bahwa di Cina, Garam digunakan
(B) To make coins untuk untuk membuat koin, jadi jawaban terbaik
(C) To build houses adalah jawaban (B).
(D) To spice food

On the recording, you hear:


(Narrator) What does “salary” mean in English?
Pertanyaan ketiga menanyakan arti dari
In your test book, you read:
"gaji". Pembicara mengatakan gaji yang
(A) Commodities
(B) Soldiers berarti jumlah uang, jadi jawaban terbaik
(C) Earnings adalah jawaban (C).
(D) Coins

On the recording, you hear:


(Narrator) What is the meaning of the root “sal” in Latin?
In your test book, you read:
(A) Trade Pertanyaan terakhir menanyakan tentang
(B) Money arti akar kata dari "sal". Pembicara
(C) Rome mengatakan bahwa "sal" berasal dari akar
(D) Salt bahasa Latin untuk "garam, jadi jawaban
terbaik adalah jawaban (D)

Center for International Language Development | 11


TOEFL EXERCISE 7
Listen to each complete talk and answer the questions that follow.

1. (A) sociology 3. (A) Later today


(B) economy (B) By Friday of this week
(C) geography (C) In two weeks
(D) In three weeks
(D) biology
4. (A) Journal and magazine articles
2. (A) On the first day of class (B) Books from outside the library
(B) In the middle of the semester (C) Books listed in student journals
(C) At the end of class (D) Both book and journals
(D) In the final week of the 5. (A) Two
semester (B) Three
(C) Five
(D) Seven

TOEFL EXERCISE 8
Listen to each complete talk and answer the questions that follow.
1. (A) An artist
(B) A tour guide 4. (A) The American History Museum
(C) An Indian (B) The Smithsonian Arts and
(D) Overville Wright Industries Building
(C) The Washington Museum
2. (A) Several
(D) The National Air and Space
(B) Sixty thousand
(C) Six million Museum
(D) Millions and millions
5. (A) To the White House
3. (A) The National Air and Space (B) To the Smithsonian
Museum (C) To the mall
(B) The Museum of Natural History (D) To various other museum
(C) The American History Museum
(D) The Smithsonian Arts and
Industries Building

Center for International Language Development | 12


REVIEW LISTENING COMPREHENSION

PART A
Directions: In Part A you will hear short conversations between two people. After each conversation,
you will hear a question about the conversation. The conversations and questions will not be repeated.
After you hear a question, read the four possible answers in your test book and choose the best
answer. Then on your answer sheet, find the number of the question and fill in the space that
corresponds to the letter of the answer you have chosen.

1. (A) He‟s feeling better today than yesterday. 8. (A) There were two exams rather than
(B) He‟s not feeling very well. one.
(C) He‟d really rather not answer. (B) The exam was not very difficult.
(D) He‟s rather sick of working the question. (C) He thought the exam would be easier.
(D) The final exam was harder than the
2. (A) In a beauty salon
others.
(B) In a bank
(C) In restaurant 9. (A) A party caterer
(D) At a service station (B) A salesclerk in a shoe store
(C) A salesclerk in a fixtures department
3. (A) He thought the lesson didn‟t matter.
(D) A shoe repair person
(B) He didn‟t like most of the lesson.
(C) He couldn‟t learn the lesson. 10. (A) The man said something foolish.
(D) He learned a massive number of details. (B) She agrees that the food is pretty bad.
(C) She thinks that the food is the best she
4. (A) The trip would cost too much.
has ever tasted.
(B) She doesn‟t think that a trip would be
(D) The man should repeat what he said.
a good idea.
(C) She would like to take two trips 11. (A) She has to wait for some cash
rather than one. (B) She needs to watch out for a crash
(D) She would also like to take a trip. (C) The lawn is too dry
(D) The waiter is bringing a glass of water
5. (A) He would like to see the elections for
town council. 12. (A) The sweet girl doesn‟t feel right
(B) He disagrees with the woman. (B) The man‟s feet aren‟t sweating
(C) He thinks the elections should take (C) The sweater‟s the wrong size
place next month. (D) The sweater makes the man seem fat
(D) He agrees that matt should be elected.
13. (A) She‟s rather go running
6. (A) He‟s flight attendant. (B) She needs a sweat suit to go running
(B) He‟s a member of the ground crew. (C) She doesn‟t want to go into the pool
(C) He works clearing land. (D) She‟ll change clothes quickly and go
(D) He‟s a pilot. swimming
7. (A) This party hasn‟t been any fun at all. 14. (A) The plane landed in the right place.
(B) He wonders if the woman enjoyed (B). He had to wait for the plane to land
herself. (C) The plane arrived on time
(C) He‟s enjoyed himself tremendously. (D) He did not look at the right schedule.
(D) He wants to know what he said.

Center for International Language Development | 13


15. (A) She is not sure which course she 18. (A) Wash the dishes for as long as
should take possible
(B) Knows that she is not ready for (B) Wait until later to clean up
intermediate French (C) Use as many dishes as possible
(C) She wants to take neither beginning (D) Wash the dishes immediately
nor intermediate French
19. (A) It was just established that he could
(D) She‟s not sure if she should take a trip
go into business
to France
(B) The family is well-established
16. (A) Some animals started the first fire (C) The business only lasted a year.
(B) In the first frost, animals die (D) The company was founded about a
(C) Frost can kill animals year ago
(D) Animals are killed by forest fires
20. (A) He regularly goes to communities
17. (A) In a biology laboratory around Boston.
(B) In a photography studio (B) He has been regularly using computer.
(C) In the library (C) He communicates with a Boston
(D) In an office company.
(D) He has been traveling back and forth
to Boston.

PART B
Directions: In this part of the test, you will hear longer conversations. After each conversation, you
will hear several questions. The conversations and questions will not be repeated.
After you hear a question, read the four possible answers in your test book and choose the best
answer. Then, on your answer sheet, find the number of the question and fill in the space that
corresponds to the letter of the answer you have chosen.

21. (A) After the haircut, Bob's hair still 26. (A) See a play
touches the floor. (B) Eat in the cafeteria and study
(B) This is Bob's first haircut. (C) Go to a sporting event
(C) Bob doesn't know who gave him the (D) Attend a football game alone
haircut.
27. (A) It‟s a very important game.
(D) The haircut is unusually short.
(B) It‟s better than the drama department‟s
22. (A) He dislikes it immensely. play.
(B) It is just what he wanted. (C) It‟s the final game of the season.
(C) He enjoys having the latest style. (D) It‟s close to the cafeteria.
(D) He thinks it will be cool in the
28. (A) At the campus
summer.
(B) At the music department
23. (A) To cut all of his hair (C) At the drama department
(B) To trim his hair just a little bit (D) At the cafeteria
(C) To go to town
29. (A) A game
(D) Put his hair on the floor
(B) A play
24. (A) Broken mirror (C) Dinner in the cafeteria
(B) Piles of his hair (D) A study group meeting
(C) The hairstylist
30. (A) After dinner in the cafetaria
(E) The scissors used to cut his hair
(B) Saturday night
25. (A) "You should become a hairstylist." (C) Maybe next weekend
(B) "Please put it back on." (D) Sunday afternoon
(C) "It won't grow fast enough."
(D) "It'll grow back."

Center for International Language Development | 14


PART C
Directions: In this part of the test, you will hear several talks. After each talk, you will hear some
questions. The talks and questions will not be repeated.
After you hear a question, read the four possible answers in your test book and choose the best
answer. Then, on your answer sheet, find the number of the question and fill in the space that
corresponds to the letter of the answer you have chosen.

31. (A) Overwille Wright


(B) An artist
(C) A tour guide
(D) An Indian
32. (A) Millions and millions
(B) Six million
(C) Sixty thousand
(D) Several
33. (A) The American History Museum
(B) The Smithsonian Arts and Industries Building
(C) The National Air and Space Museum
(D) The Museum of Natural History
34. (A) The Smithsonian Arts and Industries Building
(B) The National Air and Space Museum
(C) The American History Museum
(D) The Washington Museum
35. (A) To the Smithsonian
(B) To the White House
(C) To various other museum
(D) To the mall
36. (A) biology
(B) sociology
(C) economy
(D) geography
37. (A) In the final week of the semester
(B) On the first day of class
(C) In the middle of the semester
(D) At the end of class
38. (A) In two weeks
(B) In three weeks
(C) By Friday of this week
(D) Later today
39. (A) Books from outside the library
(B) Books listed in student journals
(C) Both book and journals
(D) Journal and magazine articles
40. (A) Seven
(B) Five
(C) Three
(D) Two

Center for International Language Development | 15


STRUCTURE & WRITTEN
EXPRESSION

Skill I: BE SURE THE SENTENCE HAS A SUBJECT AND A VERB


Dalam mengerjakan soal TOEFL, pastikan dalam sebuah kalimat terdapat subject dan verb
1
serta pastikan antara subject dan verb sesuai satu sama lain. Perhatikan contoh soal berikut:

Example 1
was backed up for miles on the freeway.

(A) Yesterday (C) Traffic


(B) In the morning (D) Cars

Kalimat di atas sudah memilki verb (kata kerja) tetapi belum memiliki subject. Diantara pilihan
jawaban tersebut yang dapat menjadi subject dan sesuai dengan kata kerja was adalah pilihan
jawaban (C) Trafic.

Perhatikan contoh ke-2:

Example 2
Tiger moths wings they have stripes or spots marks
A B C D

Pada contoh ini Anda diminta untuk memilih bagian yang salah dalam kalimat tersebut. Bagian
yang salah dalam kalimat di atas adalah (B) they karena subjectnya adalah “Tiger moths wings” yang
berpasangan dengan kata kerja “have” sehingga kata “they” tidak diperlukan.

EXERCISE 1:
Choose the letter of the word or group of words that best completes the sentence.

1. The North Pole __________ a latitude of 90 3. ____________range in color from pale


degrees north. yellow to bright orange.
(A) it has (A) canaries
(B) is having
(B) canaries which
(C) which is having
(D) has (C) that canaries
(D) canaries that are
2. A pride of lions___________up to forty lions,
including one of three males, several females, 4. The musical instrument______________
and cubs. (A) it is called the bass
(A) contain (B) is called the bass
(B) it contains (C) calling the bass
(C) contains (D) call the bass
(D) containing

Center for International Language Development | 16


5. ____________reached the North Pole on
April 6, 1909.
(A) he made
(B) Robert Peary
(C) his
(D) him

EXERCISE 2:
Choose the letter of the underlined word or group of words that is not correct.

1. In Japan, student get free lunch from school.


A B C D

2. The fir trees were grown for the holiday season were harvested in November.
A B C D

3. A specialty shop are in the shopping mall.


A B C D

4. Operas can be broadly classified as either comedies or they are tragedies.


A B C D

5. The great digital advances, such as integrated circuitry and a microcomputer, has been

2
A B C
planted in tiny chips.
D

Skill 2: BE CAREFUL OF OBJECTS OF PREPOSITIONS

Object of preposition adalah noun atau pronoun yang muncul setelah preposition misalnya in,
at, of, to, by behind, on, dan lain lain,. Object of preposition tidak dapat menjadi subjek dalam
kalimat.
(After his exams) Tom will take a trip (by boat)

Kalimat di atas memiliki dua object of preposition. Exams adalah object of preposition dari after dan
boat adalah object of preposition dari by. Perhatikan contoh soal berikut:

Example 1
With his friend found the movie theater.
(A) has (C) later
(B) he (D) when

“His friend” dalam soal di atas tidak dapat menjadi subjek karena merupakan objek dari preposisi
“with”. Subjek dalam soal di atas adalah (B) he.

Center for International Language Development | 17


Perhatikan contoh soal ke-2:

Example 2
In the United States, water treatment are generally the responsibility of municipal governments.
A B C D

Subjek dalam soal tersebut adalah water treatment yang memerlukan kata kerja untuk subjek tunggal
sehingga bagian yang salah adalah (B) are.

EXERCISE 3:
Choose the letter of the word or group of words that best completes the sentence!

1. The major cause______________the pull 4. The report with complete documentations


of the Moon and the Earth ___________ the conference
(A) the ocean tides are (A) which are delivered
(B) of ocean tides is (B) was delivered at
(C) of the tides in the ocean (C) delivering
(D) the ocean‟s tides (D) deliver
2. _________ of any new ideas for future 5. The progressive reading methods at this
products has to be approved in advance school ____________ the improved test
(A) The latest development scores.
(B) After the development (A) give credits
(C) During the development (B) is given credits for
(D) Several developments (C) gave credits
(D) were given credits for
3. The interviews __________were carried
live by the station.
(A) were the broadcasters.
(B) the broadcaster.
(C) by the broadcaster.
(D) are broadcasted.

EXERCISE 4:
Choose the letter of the underlined word or group of words that is not correct.

1. The tenants in the apartment next to mine is giving a party this evening.
A B C D

2. Manufacture of the items that you requested have been discontinued because of lack of
A B C
profit on those items.
D

3. The scheduled departure time of the trains, posted on panels throughout the terminal
A B
buildings, are going to be updated.
C D

Center for International Language Development | 18


4. The building destroyed during the fire are being rebuilt at the taxpayers' expense.
A B C D

5. For the last three years at various hospitals in the country have been practicing medicine.
A B

Skill 3: MAKE VERBS AGREE AFTER CERTAIN WORDS


C D
3
SUBJECT/VERB AGREEMENT AFTER CERTAIN WORDS

Kata-kata di bawah ini dalam Bahasa Inggris selalu dianggap tunggal, walaupun mungkin diikuti
oleh kata benda jamak:

anybody everybody nobody somebody each(+noun)anyone


anything everyone no one someone every(+ noun)
anyone everything nothing something

Perhatikan contoh soal berikut:


Example 1
Everybody __________to the theater.
(A) are going (C) go
(B) is going (D) going

Jawaban yang tepat untuk soal tersebut adalah (B) is going karena subjek Everybody selalu
dianggap tunggal dan memerlukan kata kerja tunggal.

Perhatikan contoh soal ke-2:

Example 2
Every items in this house are bought with my own money
A B C D

Subjek items pada soal tersebut dianggap tunggal karena diawali dengan kata every sehingga kata
kerja yang sesuai adalah kata kerja untuk subjek tunggal. Bagian yang salah dalam soal di atas adalah
(B) are karena kata kerja are digunakan untuk subjek jamak.

EXERCISE 5
Choose the letter of the word or group of words that best completes the sentence!
1. The company reiterated to reporters that 2. It is nice to believe that anything _________,
nobody __________because of the incident. if a person tries hard enough.
(A) has been dismissed (A) possibly
(B) are dismissed (B) quite possible
(C) have been dismissed (C) is possible
(D) are being dismissed (D) impossible

Center for International Language Development | 19


3. Each of the doctors in this building (B) Breaks are taken
_________to have separate reception area. (C) takes unnecessary breaks
(A) Are needing (D) have taken a break
(B) Are needed
(C) Needs 5. It is impossible to believe that somebody
(D) Need actually ___________that man.
(A) Admires
4. Every time someone ______________, (B) Have admires
precious moments of production time is lost. (C) Admire
(A) Are taking breaks unnecessarily (D) Have been admiring

EXERCISE 6
Choose the letter of the underlined word or group of words that is not correct.

1. Each package that is not properly wrapped have to be returned to the sender.
A B C D
2. Everybody participating in the fund-raiser are to turn in the tickets by 8:00.
A B C D
3. Because of the low number of orders, nothing have to be done now.
A B C D
4. Every man, woman, and child in this line are required to sign the forms in order
A B
to complete the registration process.
C D
5. Anybody who goes to the top of the Empire State Building are impressed with the view.
A

Skill 4: AFTER HAVE, USE THE PAST PARTICIPLE


B C D
4
Ketika Anda menemukan kata kerja bantu have/has/had/having, pastikan kata kerja yang
mengikutinya adalah kata kerja bentuk ke-3 (past participle).

Verb forms after have


HAVE + PAST PARTICPLE
My friend has sung in the choir.
We have seen the show
Having eaten, he went to school.

Perhatikan contoh soal berikut ini:


Example 1
Having_________the term paper, she began studying for the exam.
(A) Finishing (C) Finishes
(B) Finish (D) Finished

Center for International Language Development | 20


Jawaban yang tepat untuk soal di atas adalah (D) finished karena di awali dengan kata having
sehingga memerlukan kata kerja bentuk ke-3 (past participle).

Example 2
Before the time was up, all the test takers had leaves the room
A B C D

Bagian yang salah dari kalimat di atas adalah (D). Kata leaves seharusnya dalam bentuk past
participle (left) karena muncul setelah had.

EXEXERCISE 7
Choose the letter of the word or group of words that best completes the sentence!

1. Before she left, she_________her mother (C) had seeing


for permission (D) have never see
(A) had asked 4. The thief simply walked in.
(B) has ask Someone_____________to lock the door.
(C) have asking (A) had forgotten
(D) had asks
(B) had forgot
2. You might ______________more (C) forgotten
effectively (D) have had forget
(A) has responded 5. The patient_____________in the
(B) have responded emergency room for almost an hour before
(C) has responding a doctor finally treated him.
(D) have responds
(A) had waiting
3. I ___________any of Picasso‟s paintings (B) have wait
before I visited the art museum. (C) had been waiting
(A) had saw (D) had been wait
(B) had never seen

EXERCISE 8
Choose the letter of the underlined word or group of words that is not correct.

1. I have gave you all the money that I have.


A B C D
2. He thought that he should have be invited to attend the conference.
A B C D
3. Tom have thought about taking that job
A B C D
4. The secretary has recently broke her typewriter.
A B C D
5. He has often became angry during meetings.
A B C D

Center for International Language Development | 21


Skill 5 : AFTER WILL, WOULD, OR OTHER MODALS, USE THE BASE FORM OF
THE VERB

VERB FORMS AFTER MODALS


MODAL + base form of the verb
Ketika Anda menemukan modal seperti: will, would, shall, should, can, could, may, might, dan must,
pastikan kata kerja yang mengikutinya adalah kata kerja bentuk dasar. Contohnya:
5
The boat will leave at 3:00.
The doctor may arrive soon

Perhatikan contoh soal berikut ini:

Example 1:
The company will_____________the salary of each employees.
(A) raise (C) raised
(B) raising (D) be raising

Jawaban yang tepat untuk soal tersebut adalah (A) raise karena modal will selalu diikuti oleh kata
kerja bentuk dasar. Perhatikan contoh soal kedua:

Example 2:

Every morning the plants must been watered.


A B C D
Bagian yang salah dari kalimat di atas adalah (C) been karena setelah modal must selalu diikuti
oleh kata kerja bentuk dasar. Kalimat yang benar menjadi Every morning the plants must be watered.

EXERCISE 9
C Choose the letter of the word or group of words that best completes the sentence!

1. Bioluminescent animal can_________the (A) he can have


water or on a land. (B) she have
(A) they are found in (C) she can have
(B) lives (D) can have
(C) be found in
4. The installer ___________the task more
(D) are living either
quickly.
2. If Handoyo had really wanted to pass his (A) shall been completes
exam, he___________much more. (B) should have completed
(A) would has studied (C) will being complete
(B) would have studied (D) will having to be complete
(C) would studied
5. Should I ____________ you another cup of
(D) would studying
coffee?
3. She will work on the project only if (A) bringing
___________a full time secretary. (B) brings
(C) has brought
(D) bring

Center for International Language Development | 22


EXERCISE 10
Choose the letter of the underlined word or group of words that is not correct.

1. The report should had been submitted by noon.


A B C D

2. He could has been taking four courses this semester.


A B C D

3. The score information has been duplicates on the back-up disk


A B C D

4. The houses with ocean views could sold for considerably more
A B C D

5. The machine may clicks off if it is overused


A B CD

Center for International Language Development | 23


REVIEW STRUCTURE & WRITTEN EXPRESSION

Choose the letter of the word or group of words that best completes the sentence!

1. The installer ___________the task more 7. The patient_____________in the


quickly. emergency room for almost an hour before
(A) shall been completes a doctor finally treated him.
(B) will having to be complete (A) had been waiting
(C) will being complete (B) had been wait
(D) should have completed (C) had waiting
(D) have wait
2. Should I ____________ you another cup
of coffee? 8. A pride of lions___________up to forty
(A) bring lions, including one of three males, several
(B) bringing females, and cubs.
(C) bring (A) contains
(D) has brought (B) containing
(C) contain
3. I ___________any of Picasso‟s paintings
(D) it contains
before I visited the art museum.
(A) had never seen 9. _________ of any new ideas for future
(B) had saw products has to be approved in advance
(C) had seeing (A) Several developments
(D) have never see (B) During the development
(C) After the development
4. The progressive reading methods at this
(D) The latest development
school ____________ the improved test
scores. 10. It is nice to believe that anything
(A) gave credits _________,if a person tries hard enough.
(B) is given credits for (A) impossibly
(C) give credits (B) is possible
(D) were given credits for (C) quite possible
(D) possible
5. The North Pole __________ a latitude of 90
degrees north. 11. Before she left, she_________her mother
(A) has for permission
(B) which is having (A) has ask
(C) is having (B) had asked
(D) it has (C) have asking
(D) had asks
6. The major cause______________the pull
of the Moon and the Earth 12. Bioluminescent animal can_________the
(A) the ocean tides are water or on a land.
(B) of the tides in the ocean (A) they are found in
(C) of ocean tides is (B) be found in
(D) the ocean‟s tides (C) lives
(D) are living either

Center for International Language Development | 24


13. _____________range in color from pale 20. Every time someone ______________,
yellow to bright orange. precious moments of production time is lost.
(A) that canaries (A) have taken a break
(B) canaries which (B) breaks are taken
(C) Canaries (C) takes unnecessary breaks
(D) canaries that are (D) are taking breaks unnecessarily
14. The interviews __________were carried 21. The report with complete documentations
live by the station. ___________ the conference
(A) were the broadcasters. (A) which are delivered
(B) are broadcasted (B) delivering
(C) by the broadcaster. (C) was delivered at
(D) the broadcaster (D) deliver
15. Each of the doctors in this building 22. The musical instrument______________
_________to have separate reception area. (A) calling the bass
(A) are needed (B) is called the bass
(B) are needing (C) it is called the bass
(C) need (D) call the bass
(D) needs
23. The thief simply walked in.
16. The company reiterated to reporters that Someone_____________to lock the door.
nobody __________because of the (A) had forgot
incident. (B) had forgotten
(A) has been dismissed (C) have had forget
(B) are dismissed (D) forgotten
(C) have been dismissed
24. ____________reached the North Pole on
(D) are being dismissed
April 6, 1909.
17. She will work on the project only if (A) His
___________a full time secretary. (B) Him
(A) can have (C) He made
(B) she can have (D) Robert Peary
(C) she have
25. It is impossible to believe that somebody
(D) he can have
actually___________that man.
18. If Handoyo had really wanted to pass his (A) admire
exam, he___________much more. (B) have been admiring
(A) would have studied (C) admires
(B) would has studied (D) has been admires
(C) would studying
(D) would studied
19. You might ______________more
effectively
(A) have responded
(B) has responded
(C) have responds
(D) has responding

Center for International Language Development | 25


Choose the letter of the underlined word or group of words that is not correct.

1. The machine may clicks off if it is overused


A B CD
2. He has often became angry during meetings.
A B C D
3. Anybody who goes to the top of the Empire State Building are impressed with the view.
A B C D
4. For the last three years at various hospitals in the country have been practicing medicine.
A B C D
5. The great digital advances, such as integrated circuitry and a microcomputer, has been
A B C
planted in tiny chips.
D
6. In Japan, student get free lunch from school.
A B C D
7. The tenants in the apartment next to mine is giving a party this evening.
A B C D
8. Each package that is not properly wrapped have to be returned to the sender.
A B C D
9. I have gave you all the money that I have.
A B C D
10. The report should had been submitted by noon.
A B C D
11. A specialty shop are in the shopping mall.
A B C D
12. Manufacture of the items that you requested have been discontinued because of lack of
A B C
profit on those items.
D
13. The secretary has recently broke her typewriter.
A B C D
14. Everybody participating in the fund-raiser are to turn in the tickets by 8:00.
A B C D
15. Tom have thought about taking that job
A B C D

Center for International Language Development | 26


16. He has often became angry during meetings.
A B C D
17. The houses with ocean views could sold for considerably more
A B C D
18. Operas can be broadly classified as either comedies or they are tragedies.
A B C D
19. Because of the low number of orders, nothing have to be done now.
A B C D
20. Every man, woman, and child in this line are required to sign the forms in order
A B
to complete the registration process.
C D
21. The building destroyed during the fire are being rebuilt at the taxpayers' expense.
A B C D
22. He could has been taking four courses this semester.
A B C D
23. The score information has been duplicates on the back-up disk
A B C D
24. The scheduled departure time of the trains, posted on panels throughout the terminal
A B
buildings, are going to be updated.
C D
25. He thought that he should have be invited to attend the conference.
A B C D

Center for International Language Development | 27


READING COMPREHENSION
Skill 1: ANSWER MAIN IDEAQUESTIONS CORRECTLY

Almost every reading passage on the TOEFL test will have a


1
question about the main idea of a passage. You will be asked to
identify the topic, subject, title, primary idea, or main idea.
If a passage consists only one paragraph, you should study the
beginning of that paragraph to determine the main idea.

Example I
The passage:
In the philosophy of John Dewey, a sharp distinction is made between “intelligence” and
“reasoning.” According to Dewey, intelligence is the only absolute way to achieve a balance
between realism and idealism, between practically and wisdom of life. Intelligence involves
“interacting with other things and knowing them,” while reasoning is merely the act of an
observer, “... a mind that beholds or grasps object outside the world of things...”
With reasoning, a level of mental certainty can be achieved, but it is through intelligence that
control is taken of events that shape one‟s life.
The question:
What is the topic of this passage?
(A) Distinctions made by John Dewey
(B) The intelligence of John Dewey
(C) How intelligence differs from reasoning in Dewey‟s works
(D) Dewey‟s ideas on the ability to reason

The best answer is therefore (C); the idea of how intelligence differs from reasoning comes from the
first sentence of the passage, which mentions a sharp distinction ... between “intelligence” and
“reasoning.”
If a passage consists of more than one paragraph, you should study at the beginning of each paragraph
to determine the main idea.

MAIN IDEA QUESTIONS


HOW TO IDENTIFY THE  What is the topic of the passage?
QUESTION  What is the subject of the passage?
 What is the main idea of the passage?
 What is the author‟s main point in the passage?
 With what is the author primarily concerned?
 Which of the following would be the best title?
WHERE TO FIND THE ANSWER The answer to this type of question can generally be
determined by looking at the first sentence of each
paragraph.

Center for International Language Development | 28


HOW TO ANSWER THE 1. Read the first line of each paragraph.
QUESTION 2. Look for a common theme or idea in the first lines.
3. Pass your eyes quickly over the rest of the passage to
check that you really have found the topic sentence(s).
4. Eliminate any definitely wrong answers and choose
the best answer from the remaining choices.

TOEFL EXERCISE 1

Fort Knox, Kentucky, is the site of a U.S. army post, but it is even more renowned for the Fort
Knox Bullion Depository, the massive vault that contains the bulk of the U.S. government‟s gold
deposits. Completed in 1936, the vault is housed in a two-story building constructed of granite, steel,
and concrete; the vault itself is made of steel and concrete and has a door that weighs more than
twenty tons. Naturally, the most up-to-date security devices available are in place at Fort Knox, and
the army post nearby provides further protection

1. Which of the following best describes the 2. Which of the following would be the best
topic of the passage? title for this passage?
(A) Gold bullion (A) Where the U.S. Keeps Its Gold
(B) The city of Fort Knox Kentucky (B) A Visit to Kentucky
(C) The U.S. army post at Fort Knox (C) The Massive Concrete Vault
(D) The federal gold depository (D) Fort Knox Security

One identifying characteristic of minerals is their relative hardness, which can be determined
by scratching one mineral with another. In this type of test, a harder mineral can scratch a softer one,
but a softer mineral is unable to scratch the harder one. The Mohs‟ scale hardness scale is used to rank
minerals according hardness. Ten minerals are listed in this scale, ranging from talc with a hardness
of 1 to diamond with a hardness of 10. On this scale, quartz (number 7) is harder than feldspar
(number 6) and is therefore able to scratch it; however feldspar is unable to make a mark on quartz.

3. Which of the following best states the 4. Which of the following would be the best
subject of this passage? title for this passage?
(A) Feldspar on the Mohs‟ scale (A) Talc is the first mineral listed on the
(B) The hardness of diamonds. Mohs‟ scale
(C) Identifying minerals by means of a (B) A softer mineral cannot be scratched
scratch test by a harder mineral
(D) Recognizing minerals in their natural (C) Diamonds, with a hardness of 10 on
state the Mohs‟ scale, can scratch all other
minerals
(D) The hardness of a mineral can be
determined by its ability to make a
mark on other minerals

Center for International Language Development | 29


Skill 2: ANSWER STATED DETAIL QUESTIONS CORRECTLY
2
A stated detail question asks about one piece of information in the passage rather than the passage as a
whole.

Example
The passage:
Williamsburg is a historic city in Virginia situated on a peninsula between two rivers, the York
and the James. It was settled by English colonists in 1633, twenty-six years after the first
permanent English colony in America was settled at Jamestown. In the beginning the colony at
Williamsburg was named Middle Plantation because of its location in the middle of the peninsula.
The site for Williamsburg had been selected by the colonists because the soil drainage was better
three than at the Jamestown location, and there were fewer mosquitoes.
The questions:
1. According to the passage, Williamsburg is located….
(A) on an island
(B) on a piece of land with rivers on two sides
(C) where the York and James meet
(D) in the middle of a river

The best answer to the first question is answer (B); with rivers in two sides is closest in meaning to
between two rivers.

STATED DETAIL QUESTIONS


HOW TO IDENTIFY THE QUESTION  According to the passage, ...
 It is stated in the passage...
 The passage indicates that...
 The author mentions that...
 Which of the following is true...?
WHERE TO FIND THE ANSWER The answers to these questions are found in
order in the passage.
HOW TO ANSWER THE QUESTION 1. Choose a key word in the question.
2. Skim in the appropriate part of the passage
for the key word or idea.
3. Read the sentence that contains the key word
or idea carefully.
4. Look for the answer that restates an idea in
the passage.
5. Eliminate the definitely wrong answers and
choose the best answer from the remaining
choices.

Center for International Language Development | 30


TOEFL EXERCISE 2

Ice ages, those periods when ice covered extensive areas of the Earth, are known to have
occurred at least six times. Past ice ages can be recognized from rock strata that show evidence of
foreign materials deposited by moving walls of ice or melting glaciers. Ice ages can also be
recognized from land formations that have been produced from moving walls of ice, such as U-shaped
valleys, sculptured landscapes, and polished rock faces

1. According to the passage, what happens 3. According to the passage, what in the rock
during an ice age? strata is a clue to geologists of a past ice age?
(A) Ice melts six times. (A) Substances from other areas
(B) Ice covers a large portion of the Earth‟s (B) U-shaped valleys
surface. (C) Melting glaciers
(C) Evidence of foreign materials is found (D) Ice
(D) Rock strata are recognized by
Geologists

2. The passage covers how many different


methods of recognizing past ice ages?
(A) Three

3
(B) Four
(C) One
(D) Two

Skill 3: FIND PRONOUN REFERENTS

Whenever you are asked which noun a pronoun refers to, you should look before the pronoun to find
the noun

Example
The passage:
Carnivorous plants, such as the sundew and the Venus-flytrap, are generally found in humid
areas where there is an inadequate supply of nitrogen in the soil. In order to survive, these plants
have developed mechanism to trap insects within their foliage. They have digestive fluids to
obtain the necessary nitrogen from the insects. These plants trap the insects in a variety ways.
The sundew has sticky hairs on its leaves; when an insect lands on these leaves, it gets caught up
in the sticky hairs, and the leaf wraps itself around the insect. The leaves of the Venus-fly trap
function more alike a trap, snapping suddenly and forcefully shut around an insect
The questions:
1. The pronoun “they” in line 4 refers to….
(A) Humid areas
(B) insects
(C) these plants
(D) digestive fluids

In the first example, you should understand from the context that these plants have digestive fluids to
obtain the necessary nitrogen from the insects, so the best answer to this question is answer (C).

Center for International Language Development | 31


PRONOUN REFERENTS
HOW TO IDENTIFY THE QUESTION The pronoun “...” in line X refers to which of the
following?
WHERE TO FIND THE ANSWER The line where the pronoun is located is generally
given in the question. The noun that the pronoun
refers to is generally found before the pronoun.

HOW TO ANSWER THE QUESTION 1. Find the pronoun in the passage. (The line where
the pronoun can be found is generally stated in
the question.)
2. Look for nouns that come before the pronoun.
3. Read the part of the passage before the pronoun
carefully.
4. Eliminate any definitely wrong answers and
choose the best answer from the remaining
choices.

TOEFL EXERCISE 3

The full moon that occurs nearest the equinox of the Sun has become known as the harvest
moon. It is a bright moon which allows farmers to work late into the night for several nights; they can
work when the moon is at its brightest to bring in the fall harvest. The harvest moon of course occurs
at different times of the year in the northern and southern hemisphere, the harvest moon occurs in
September at the time of the autumnal equinox. In the southern hemisphere, the harvest moon occurs
in March at the time of the vernal equinox.

1. The pronoun “It” in line 2 refers to…. 2. The pronoun “they” in line 3 refers to….
(A) The night (A) times of the year
(B) the harvest moon (B) nights
(C) the Sun (C) farmers
(D) the equinox (D) northern and southern hemisphere

Center for International Language Development | 32


READING EXERCISES

Questions 1 – 5.

Most people think of deserts as dry, flat areas with little vegetation and little or no rainfall,
but this is hardly true. Many deserts have varied geographical formations ranging from soft, rolling
hills to stark, jagged cliffs, and most deserts have a permanent source of water. Although deserts do
not receive a high amount of rainfall —to be classified as a desert, an area must get less than twenty-
five centimeters of rainfall per year there are many plants that thrive on only small amounts of
water, and deserts are often full of such plant life.
Desert plants have a variety of mechanisms for obtaining the water needed for survival.
Some plants, such as cactus, are able to store large amounts of water in their leaves or stems; after a
rainfall these plants absorb a large supply of water to last until the next rainfall. Other plants, such as
the mesquite, have extraordinarily deep root systems that allow them to obtain water from far below
the desert's arid surface.

1. What is the main topic of the passage?


(A) Deserts are dry, flat areas with few plants.
(B) There is little rainfall in the desert.
(C) Many kinds of vegetation can survive with little water.
(D) Deserts are not really flat areas with little plant life.
2. According to the passage, what causes an area to be classified as a desert?
(A) The type of plants
(B) The geographical formations
(C) The amount of rainfall
(D) The source of water
3. The word "source" in line 3 means….
(A) supply
(B) storage space
(C) need
(D) lack
4. Which of the following is mentioned in the passage about cactus?
(A) They have deep root systems.
(B) They retain water from one rainfall to the next.
(C) They survive in the desert because they do not need water.
(D) They get water from deep below the surface of the desert
5. The word "thrive" in line 5 means….
(A) suffer
(B) grow well
(C) minimally survive
(D) decay

Center for International Language Development | 33


Questions 6 – 10.

Carbon tetrachloride is a colorless and inflammable liquid that can be produced by


combining carbon disulfide and chlorine. This compound is widely used in industry today because of
its effectiveness as a solvent as well as its use in the production of propellants.
Despite its widespread use in industry, carbon tetrachloride has been banned for home use.
In the past, carbon tetrachloride was a common ingredient in cleaning compounds that were used
throughout the home, but it was found to be dangerous: when heated, it changes into a poisonous gas
that can cause severe illness and even death if it is inhaled. Because of this dangerous characteristic,
the United States revoked permission for the home use of carbon tetrachloride in 1970. The United
States has taken similar action with various other chemical compounds.

6. The main point of this passage is that….


(A) carbon tetrachloride can be very dangerous when it is heated
(B) the government banned carbon tetrachloride in 1970
(C) although carbon tetrachloride can legally be used in industry, it is not allowed in home
products
(D) carbon tetrachloride used to be a regular part of cleaning compounds
7. According to the passage, before 1970 carbon tetrachloride was….
(A) used by itself as a cleanser
(B) banned in industrial use
(C) often used as a component of cleaning products
(D) not allowed in home cleaning products
8. The word "banned" in line 4 is closest in meaning to….
(A) forbidden
(B) allowed
(C) suggested
(D) instituted
9. The word "inhaled" in line 7 is closest in meaning to….
(A) warmed
(B) breathed in
(C) carelessly used
(D) blown
10. The word “its” in line 3 refers to….
(A) Carbon tetrachloride
(B) Industry
(C) Chlorine
(D) Liquid

Center for International Language Development | 34


REVIEW READING COMPREHENSION

Questions 1-5

While the bald eagle is one national symbol of the United States, it is not the only one. Uncle
Sam, a bearded gentleman costumed in the red, white, and blue stars and stripes of the nation's flag
is another well-known national symbol. According to legend, this character is based on Samuel
Wilson, the owner of a meat-packing business in Troy, New York.
During the War of 1812, Sam Wilson's company was granted a government contract to supply
meat to the nation's soldiers; this meat was supplied to the army in barrels stamped with the initials
U.S., which stood for United States. However, the country was at that time relatively young, and the
initials U.S. were not commonly used. Many people questioned what the initials represented, and the
standard reply became "Uncle Sam," for the owner of the barrels. It is now generally accepted that
the figure of Uncle Sam is based on Samuel Wilson, and the U.S. Congress has made it official by
adopting a resolution naming Samuel Wilson as the inspiration for Uncle Sam.

1. Which of the following is the most appropriate title for this passage?
(A) The Bald Eagle
(B) The Symbols of the United States
(C) Samuel Wilson
(D) Uncle Sam—Symbol of the Nation
2. Which of the following is NOT mentioned about Uncle Sam's appearance?
(A) He wears facial hair.
(B) There is some blue in his clothing.
(C) He is bald.
(D) His clothes have stripes in them.
3. The word "costumed" in line 2 could most easily be replaced by….
(A) dressed
(B) nationalized
(C) hidden
(D) seen
4. The pronoun “it” in line 1 refers to….
(A) The bald eagle
(B) National symbol of the United States
(C) Uncle Sam
(D) Samuel Wilson
5. According to the passage, what was in the barrels stamped U.S.?
(A) Sam Wilson
(B) Food for the army
(C) Weapons to be used in the war
(D) Company contracts

Center for International Language Development | 35


Questions 6-10

The hippopotamus is the third largest land animal, smaller only than the elephant and the
rhinoceros. Its name comes from two Greek words which mean "river horse." The long name of this
animal is often shortened to the easier to handle term "hippo."
The hippo has a natural affinity for the water. It does not float on top of the water; instead, it
can easily walk along the bottom of a body of water. The hippo commonly remains underwater for
three to five minutes and has been known to stay under for up to half an hour before coming up for
air.
In spite of its name, the hippo has relatively little in common with the horse and instead has a
number of interesting similarities in common with the whale. When a hippo comes up after a stay at
the bottom of a lake or river, it releases air through a blowhole, just like a whale. In addition, the
hippo resembles the whale in that they both have thick layers of blubber for protection and they are
almost completely hairless.

6. The topic of this passage is….


(A) the largest land animals
(B) the derivations of animal names
(C) the characteristics of the hippo
(D) the relation between the hippo and the whale
7. According to the passage, what is the maximum time that hippos have been known to stay
underwater?
(A) Three minutes
(B) Five minutes
(C) Thirty minutes
(D) Ninety minutes
8. The word "float" in line 4 is closest in meaning to….
(A) sink
(B) drift
(C) eat
(D) flap
9. The possessive "Its" in line 2 refers to….
(A) hippopotamus
(B) elephant
(C) rhinoceros
(D) horse
10. The passage states that the hippo does not….
(A) like water
(B) resemble the whale
(C) have a protective coating
(D) have much hair

Center for International Language Development | 36


Questions 11-15

People are often surprised to learn just how long some varieties of trees can live. If asked to
estimate the age of the oldest living trees on Earth, they often come up with guesses in the
neighborhood of two or perhaps three hundred years. The real answer is considerably larger than
that, more than five thousand years.
The tree that wins the prize for its considerable maturity is the bristlecone pine of California.
This venerable pine predates wonders of the ancient world such as the pyramids of Egypt, the
Hanging Gardens of Babylon, and the Colossus of Rhodes. It is not nearly as tall as the giant
redwood that is also found in California, and, in fact, it is actually not very tall compared with many
other trees, often little more than five meters in height. This relatively short height may be one of the
factors that aid the bristlecone pine in living to a ripe old age—high winds and inclement weather
cannot easily reach the shorter trees and cause damage. An additional factor that contributes to the
long life of the bristlecone pine is that this type of tree has a high percentage of resin, which prevents
rot from developing in the tree trunk and branches.

11. The best title for this passage would be….


(A) The Size of the Bristle Cone Pine
(B) Three-Hundred-Year-Old Forests
(C) The Wonders of the Ancient World
(D) An Amazingly Enduring Tree
12. According to the passage, approximately how old are the oldest trees on Earth?
(A) Two hundred years old
(B) Three hundred years old
(C) Five hundred years old
(D) Five thousand years old
13. The word "estimate" in line 2 is closest in meaning to….
(A) measure
(B) approximate
(C) evaluate
(D) view
14. The pronoun “they” in line 2 refers to….
(A) Living trees on earth
(B)Trees
(C)The age of the trees
(D) People
15. The pronoun “it” in line 7 refers to….
(A) the pyramids of Egypt
(B)the bristlecone pine of California
(C)the Hanging Gardens of Babylon
(D) the Colossus of Rhodes

Center for International Language Development | 37


Questions 16-20

Venus is always considered as Earth‟s sister planet since both of them share almost similar
size, mass, and thin atmosphere. However unlike Earth, Venus has scorching surface. Data from the
Pioneer spacecraft of NASA apparently prove the theory that the high surface temperature of Venus
is due to an atmospheric greenhouse effect caused mainly by a blanket of carbon dioxide. Such a
greenhouse effect is created when energy in the form of sunlight easily passes through a planet's
atmosphere, warms its surface, and is converted to heat radiation that is then held in by the
atmosphere from top to bottom.

16. The passage above mainly discusses…..


(A) Earth‟s atmosphere
(B) Earth„s Greenhouse effect
(C) Venus‟ hot temperature
(D) Venus‟s sister planet
17. According to the passage Venus…
(A) Is bigger than Earth
(B) has higher temperature than Earth
(C) Has thick atmosphere
(D) Has cooler surface than Earth.
18. The word “them” in line 1 refers to….
(A) Similarities between Earth and Venus
(B) Atmospheres
(C) Venus
(D) Earth and Venus
19. The word “its” in line 6 refers to ….
(A) Planet
(B) Sun
(C) Venus
(D) Earth
20. The word “blanket” in line 5 can be replaced by….
(A) Sheet
(B)Bed
(C)Casing
(D) Cover

Center for International Language Development | 38


Questions 21-25

Theories about how brain works remain a topic of debate. It is agreed, though, that the
hippocampus, a part of the brain, is undeniably important for memory. When we experience
something, the information is sent via our senses to the hippocampus, where it is processed.
Scientists believe that brain cells called neurons first transform the sensory stimuli we experience
into images in our immediate memory. Then, these images are sent to the hippocampus and stored
temporarily in short term memory. In the hippocampus information is organized, and it is during this
process that parts of the image of our experience fade away. Finally, certain information is then
transferred to long term memory in a section in the frontal lobe of the brain known as the cerebral
cortex. Scientists think this process may happen while we are sleeping, but exactly how the
information is transferred from one area of the brain to, they still cannot figure it out.

21. This passage is mainly concerned with….


(A) how to improve our memory
(B) why some of the information in short term memory fades away
(C) illness that results in severe memory loss
(D) how human brain processes and stores information
22. According to the passage scientists ….
(A) know that information is sent from the long term memory to the hippocampus
(B) have found out why some of the information is lost in the hippocampus
(C) don't know exactly how the information is transferred from one area of the brain to
another
(D) agree on how the brain works
23. The word “transferred” in line 8 can be replaced by….
(A) Move
(B) Sent
(C) Store
(D) Kept
24. The pronoun “it” in line 3 refers to….
(A) Brains
(B) Theories
(C) Information
(D) Memory
25. The pronouns “they” in line 11 refers to….
(A) Brains
(B) Images
(C) Processes
(D) Scientists

Center for International Language Development | 39


TRY OUT 1

SECTION 1
LISTENING COMPREHENSION

PART A

Directions: In Part A you will hear short conversations between two people. After each conversation, you will hear a
question about the conversation. The conversations and questions will not be repeated. After you hear a question, read
the four possible answers in your test book and choose the best answer. Then on your answer sheet, find the number of
the question and fill in the space that corresponds to the letter of the answer you have chosen.

1. (A) In restaurant 6. (A) It was just established that he could go into


(B) In a bank business.
(C) In a beauty salon (B) The company was founded about a year ago.
(D) At a service station (C) The business only lasted a year.
(D) The family is well-established.
2. (A) A shoe repair person
(B) A salesclerk in a shoe store 7. (A) In a biology laboratory
(C) A salesclerk in a fixtures deparment (B) In a photography studio
(D) A party caterer (C) In the library
(D) In an office
3. (A) The man said something foolish.
(B) The man should repeat what he said. 8. (A) There were two exams rather than one.
(C) She thinks that the food is the best she has ever (B) The final exam was harder than the others.
tasted. (C) He thought the exam would be easier.
(D) She agrees that the food is pretty bad. (D) The exam was not very difficult.

4. (A) He thinks the elections should take place next 9. (A) He learned a massive number of details.
month. (B) He didn‟t like most of the lesson.
(B) He disagrees with the woman. (C) He thought the lesson didn‟t matter.
(C) He would like to see the elections for town (D) He couldn‟t learn the lesson
council.
(D) He agrees that matt should be elected. 10. (A) The lawn is too dry.
(B) She needs to watch out for a crash.
5. (A) She would also like to take a trip. (C) She has to wait for some cash .
(B) She would like to take two trips rather than one. (D) The waiter is bringing a glass of water.
(C) She doesn‟t think that a trip would be a good
idea.
(D) The trip would cost too much.

Center for International Language Development | 40


PART B
Directions: In this part of the test, you will hear longer conversations. After each conversation, you will hear several
questions. The conversations and questions will not be repeated.
After you hear a question, read the four possible answers in your test book and choose the best answer. Then, on your
answer sheet, find the number of the question and fill in the space that corresponds to the letter of the answer you have
chosen.

11. (A) The haircut is unusually short. 14. (A)Abroken mirror


(B) This is Bob's first haircut. (B)The hairstylist
(C) Bob doesn't know who gave him the haircut. (C) The scissors used to cut his hair
(D) After the haircut, Bob's hair still touches the (D) Piles of his hair
floor.
15. (A) "You should become a hairstylist."
12. (A)It is just what he wanted. (B) "Please put it back on."
(B) He enjoys having the latest style. (C) "It'll grow back."
(C) He dislikes it immensely. (D) "It won't grow fast enough."
(D) He thinks it will be cool in the summer

13. (A) To go to town


(B) To cut all of his hair
(C) To trim his hair just a little bit
(D) Put his hair on the floor

PART C
Directions: In this part of the test, you will hear several talks. After each talk, you will hear some questions. The talks
and questions will not be repeated.
After you hear a question, read the four possible answers in your test book and choose the best answer. Then, on your
answer sheet, find the number of the question and fill in the space that corresponds to the letter of the answer you have
chosen.

16. (A) sociology 19. (A) Journal and magazine articles


(B) economy (B) Books from outside the library
(C) geography (C) Books listed in student journals
(D) biology (D) Both book and journals

17. (A) On the first day of class 20. (A) Two


(B) In the middle of the semester (B) Three
(C) Five
(C) At the end of class
(D) Seven
(D) In the final week of the semester

18. (A) Later today


(B) By Friday of this week
(C) In two weeks
(D) In three weeks

Center for International Language Development | 41


SECTION 2
STRUCTURE AND WRITTEN EXPRESSION
Choose the letter of the word or group of words that best completes the sentence!

1. Every time someone ______________, precious 4. If Handoyo had really wanted to pass his exam,
moments of production time is lost. he___________much more.
(A) Are taking breaks unnecessarily (A) would has studied
(B) Breaks are taken (B) would have studied
(C) takes unnecessary breaks (C) would studied
(D) have taken a break (D) would studying

2. ____________reached the North Pole on April 6, 5. Before she left, she_________her mother for
1909. permission
(A) he made (A) had asks
(B) him (B) has ask
(C) his (C) have asking
(D) Robert Peary (D) had asked

3. _________ of any new ideas for future products has 6. A pride of lions___________up to forty lions,
to be approved in advance including one of three males, several females, and
(A) The latest development cubs.
(B) After the development (A) contain
(C) During the development (B) it contains
(D) Several developments (C) contains
(D) containing

Choose the letter of the underlined word or group of words that is not correct.

7. The tenants in the apartment next to mine is giving a party this evening.
A B C D

8. The great digital advances, such as integrated circuitry and a microcomputer, has been planted in tiny chips.
A B C D

9. Everybody participating in the fund-raiser are to turn in the tickets by 8:00.


A B C D

10. Tom have thought about taking that job


A B C D

11. Anybody who goes to the top of the Empire State Building are impressed with the view.
A B C D

12. The secretary has recently broke her typewriter.


A B C D

13. The fir trees were grown for the holiday season were harvested in November.
A B C D
14. He could has been taking four courses this semester.
A B C D

15. The score information has been duplicates on the back-up disk
A B C D

Center for International Language Development | 42


SECTION 3
READING COMPREHENSION

Directions: In this section you will read several passages. Each one is followed by a number of questions about it.
You are to choose the one best answer, (A), (B), (C), or (D), to each question. Then, on your answer sheet, find the
number of the question and fill in the space that corresponds to the letter of the answer you have chosen.

Questions 1-5

Most people think of deserts as dry, flat areas with little vegetation and little or no rainfall, but this is
hardly true. Many deserts have varied geographical formations ranging from soft, rolling hills to stark, jagged
cliffs, and most deserts have a permanent source of water. Although deserts do not receive a high amount of
rainfall —to be classified as a desert, an area must get less than twenty-five centimeters of rainfall per year there
are many plants that thrive on only small amounts of water, and deserts are often full of such plant life.

Desert plants have a variety of mechanisms for obtaining the water needed for survival. Some plants, such
as cactus, are able to store large amounts of water in their leaves or stems; after a rainfall these plants absorb a
large supply of water to last until the next rainfall. Other plants, such as the mesquite, have extraordinarily deep
root systems that allow them to obtain water from far below the desert's arid surface.

1. What is the main topic of the passage? 4. Which of the following is mentioned in the
(A) Deserts are dry, flat areas with few plants. passage about cactus?
(B) There is little rainfall in the desert. (A) They have deep root systems.
(C) Many kinds of vegetation can survive with (B) They retain water from one rainfall to the
little water. next.
(D) Deserts are not really flat areas with little plant (C) They survive in the desert because they do
life. not need water.
(D) They get water from deep below the surface
2. According to the passage, what causes an area to of the desert
be classified as a desert?
(A) The type of plants 5. The word "thrive" in line 5 means….
(B) The geographical formations (A) suffer
(C) The amount of rainfall (B) grow well
(D) The source of water (C) minimally survive
(D) decay
3. The word "source" in line 3 means….
(A) supply
(B) storage space
(C) need
(D) lack

Center for International Language Development | 43


Questions 6-10

Carbon tetrachloride is a colorless and inflammable liquid that can be produced by combining carbon disulfide
and chlorine. This compound is widely used in industry today because of its effectiveness as a solvent as well as its
use in the production of propellants.

Despite its widespread use in industry, carbon tetrachloride has been banned for home use. In the past, carbon
tetrachloride was a common ingredient in cleaning compounds that were used throughout the home, but it was
found to be dangerous: when heated, it changes into a poisonous gas that can cause severe illness and even death if
it is inhaled. Because of this dangerous characteristic, the United States revoked permission for the home use of
carbon tetrachloride in 1970. The United States has taken similar action with various other chemical compounds.

6. The main point of this passage is that…. 8. The word "banned" in line 4 is closest in meaning
(A) carbon tetrachloride can be very dangerous to….
when it is heated (A) forbidden
(B) the government banned carbon tetrachloride in (B) allowed
1970 (C) suggested
(C) although carbon tetrachloride can legally be (D) instituted
used in industry, it is not allowed in home
products 9. The word "inhaled" in line 7 is closest in meaning
(D) carbon tetrachloride used to be a regular part to….
of cleaning compounds (A) warmed
(B) breathed in
7. According to the passage, before 1970 carbon (C) carelessly used
tetrachloride was…. (D) blown
(A) used by itself as a cleanser
(B) banned in industrial use 10. The word “its” in line 3 refers to….
(C) often used as a component of cleaning (A) Carbon tetrachloride
products (B) Industry
(D) not allowed in home cleaning products (C) Chlorine
(D) Liquid

Center for International Language Development | 44


Questions 11-15

While the bald eagle is one national symbol of the United States, it is not the only one. Uncle Sam, a bearded
gentleman costumed in the red, white, and blue stars and stripes of the nation's flag,is another well-known national
symbol. According to legend, this character is based on Samuel Wilson, the owner of a meat-packing business in
Troy, New York.

During the War of 1812, Sam Wilson's company was granted a government contract to supply meat to the
nation's soldiers; this meat was supplied to the army in barrels stamped with the initials U.S., which stood for
United States. However, the country was at that time relatively young, and the initials U.S. were not commonly
used. Many people questioned what the initials represented, and the standard reply became "Uncle Sam," for the
owner of the barrels. It is now generally accepted that the figure of Uncle Sam is based on Samuel Wilson, and the
U.S. Congress has made it official by adopting a resolution naming Samuel Wilson as the inspiration for Uncle
Sam.

11. Which of the following is the most appropriate 14. The pronoun “it” in line 1 refers to….
title for this passage? (A) The bald eagle
(A) The Bald Eagle (B) National symbol of the United States
(B) The Symbols of the United States (C) Uncle Sam
(C) Samuel Wilson (D) Samuel Wilson
(D) Uncle Sam—Symbol of the Nation
15. According to the passage, what was in the barrels
12. Which of the following is NOT mentioned about stamped U.S.?
Uncle Sam's appearance? (A) Sam Wilson
(A) He wears facial hair. (B) Food for the army
(B) There is some blue in his clothing. (C) Weapons to be used in the war
(C) He is bald. (D) Company contracts
(D) His clothes have stripes in them.

13. The word "costumed" in line 2 could most easily


be replaced by….
(A) dressed
(B) nationalized
(C) hidden
(D) seen

Center for International Language Development | 45


TRY OUT 2

SECTION 1
LISTENING COMPREHENSION

PART A
Directions: In Part A you will hear short conversations between two people. After each conversation, you will hear a
question about the conversation. The conversations and questions will not be repeated. After you hear a question, read
the four possible answers in your test book and choose the best answer. Then on your answer sheet, find the number of
the question and fill in the space that corresponds to the letter of the answer you have chosen

1. (A) He regularly goes to communities around 6. (A) The sweater‟s the wrong size.
Boston. (B) The man‟s feet aren‟t sweating.
(B) He communicates with a Boston company. (C) The sweet girl doesn‟t feel right.
(C) He has been traveling back and forth to Boston. (D) The sweater makes the man seem fat.
(D) He has been regularly using computer.
7. (A) Use as many dishes as possible.
2. (A) He wants to know what he said. (B) Wash the dishes immediately.
(B) He wonders if the woman enjoyed herself. (C) Wait until later to clean up.
(C) This party hasn‟t been any fun at all. (D) Wash the dishes for as long as possible.
(D) He‟s enjoyed himself tremendously. 8. (A) He had to wait for the plane to land.
3. (A) He‟s flight attendant. (B) The plane arrived on time.
(B) He‟s a pilot. (C) The plane landed in the right place.
(C) He works clearing land. (D) He did not look at the right schedule.
(D) He‟s a member of the ground crew. 9. (A) In the first frost, animals die.
4. (A) Knows that she is not ready for intermediate (B) Frost can kill animals.
French. (C) Some animals started the first fire.
(B) She wants to take neither beginning nor (D) Animals are killed by forest fires.
intermediate French. 10. (A) She needs a sweat suit to go running.
(C) She is not sure which course she should take. (B) She‟ll change clothes quickly and go swimming.
(D) She‟s not sure if she should take a trip to France. (C) She doesn‟t want to go into the pool.
5. (A) He‟d really rather not answer. (D) She‟s rather go running.
(B) He‟s not feeling very well.
(C) He‟s feeling better today than yesterday.
(D) He‟s rather sick of working the question.

Center for International Language Development | 46


PART B
Directions: In this part of the test, you will hear longer conversations. After each conversation, you will hear several
questions. The conversations and questions will not be repeated.
After you hear a question, read the four possible answers in your test book and choose the best answer. Then, on your
answer sheet, find the number of the question and fill in the space that corresponds to the letter of the answer you have
chosen.

11. (A) Attend a football game alone 14. (A) A play


(B) Go to a sporting event (B) A game
(C) Eat in the cafeteria and study (C) A study group meeting
(D) See a play (D) Dinner in the cafetaria
15. (A) Saturday night
12. (A) It‟s the final game of the season (B) After dinner in the cafeteria
(B) It‟s better than the drama department‟s play (C) Sunday afternoon
(C) It‟s a very important game (D) Maybe next weekend
(D) It‟s close to the cafetaria

13. (A) At the cafetaria


(B) At the music department
(C) At the campus
(D) At the drama department

PART C
Directions: In this part of the test, you will hear several talks. After each talk, you will hear some questions. The talks
and questions will not be repeated.
After you hear a question, read the four possible answers in your test book and choose the best answer. Then, on your
answer sheet, find the number of the question and fill in the space that corresponds to the letter of the answer you have
chosen.

16. (A) An artist 19. (A) The American History Museum


(B) A tour guide (B) The Smithsonian Arts and Industries
(C) An Indian Building
(D) Overville Wright (C) The Washington Museum
(D) The National Air and Space Museum
17. (A) Several
(B) Sixty thousand 20. (A) To the White House
(C) Sixteen million (B) To the Smithsonian
(D) Millions and millions (C) To the mall
(D) To various other museum
18. (A) The National Air and Space Museum
(B) The Museum of Natural History
(C) The American History Museum
(D) The Smithsonian Arts and Industries
Building

Center for International Language Development | 47


SECTION 2
STRUCTURE AND WRITTEN EXPRESSION

Choose the letter of the word or group of words that best completes the sentence!

1. The major cause______________the pull of the Moon 4. The company reiterated to reporters that nobody
and the Earth __________ because of the incident.
(A) the ocean tides are (A) has been dismissed
(B) of ocean tides is (B) are dismissed
(C) of the tides in the ocean (C) have been dismissed
(D) the ocean‟s tides (D) are being dismissed
2. The report with complete documentations 5. The patient_____________in the emergency room for
___________ the conference almost an hour before a doctor finally treated him.
(A) which are delivered (A) had waiting
(B) was delivered at (B) have wait
(C) delivering (C) had been waiting
(D) deliver (D) had been wait
3. The North Pole __________ a latitude of 90 degrees 6. Should I ____________ you another cup of coffee?
north. (A) bringing
(A) it has (B) brings
(B) is having (C) has brought
(C) which is having (D) bring
(D) has

7. Manufacture of the items that you requested have been discontinued because of lack of profit on those items.
A B C D

8. Operas can be broadly classified as either comedies or they are tragedies.


A B C D

9. Every man, woman, and child in this line are required to sign the forms in order to complete
A B C
the registration process.
D

10. He thought that he should have be invited to attend the conference.


A B C D

11. The houses with ocean views could sold for considerably more
A B C D

12. He has often became angry during meetings.


A B C D

13. The building destroyed during the fire are being rebuilt at the taxpayers' expense.
A B C D

14. Each package that is not properly wrapped have to be returned to the sender.
A B C D

15. The machine may clicks off if it is overused


A B CD
Center for International Language Development | 48
SECTION 3
READING COMPREHENSION

Directions: In this section you will read several passages. Each one is followed by a number of questions about it.
You are to choose the one best answer, (A), (B), (C), or (D), to each question. Then, on your answer sheet, find the
number of the question and fill in the space that corresponds to the letter of the answer you have chosen.

Questions 1-5

The hippopotamus is the third largest land animal, smaller only than the elephant and the rhinoceros. Its
name comes from two Greek words which mean "river horse." The long name of this animal is often shortened to the
easier to handle term "hippo."
The hippo has a natural affinity for the water. It does not float on top of the water; instead, it can easily walk
along the bottom of a body of water. The hippo commonly remains underwater for three to five minutes and has been
known to stay under for up to half an hour before coming up for air.
In spite of its name, the hippo has relatively little in common with the horse and instead has a number of
interesting similarities in common with the whale. When a hippo comes up after a stay at the bottom of a lake or
river, it releases air through a blowhole, just like a whale. In addition, the hippo resembles the whale in that they both
have thick layers of blubber for protection and they are almost completely hairless.

1. The topic of this passage is…. 4. The possessive "Its" in line 2 refers to….
(A) the largest land animals (A) Hippopotamus
(B) the derivations of animal names (B) Elephant
(C) the characteristics of the hippo (C) Rhinoceros
(D) the relation between the hippo and the (D) Horse
whale
5. The passage states that the hippo does not….
2. According to the passage, what is the maximum (A) like water
time that hippos have been known to stay (B) resemble the whale
underwater? (C) have a protective coating
(A) Three minutes (D) have much hair
(B) Five minutes
(C) Thirty minutes
(D) Ninety minutes

3. The word "float" in line 4 is closest in meaning


to….
(A) sink
(B) drift
(C) eat
(D) flap

Center for International Language Development | 49


Questions 6-10

People are often surprised to learn just how long some varieties of trees can live. If asked to estimate the age
of the oldest living trees on Earth, they often come up with guesses in the neighborhood of two or perhaps three
hundred years. The real answer is considerably larger than that, more than five thousand years.
The tree that wins the prize for its considerable maturity is the bristlecone pine of California. This venerable
pine predates wonders of the ancient world such as the pyramids of Egypt, the Hanging Gardens of Babylon, and the
Colossus of Rhodes. It is not nearly as tall as the giant redwood that is also found in California, and, in fact, it is
actually not very tall compared with many other trees, often little more than five meters in height. This relatively
short height may be one of the factors that aid the bristlecone pine in living to a ripe old age—high winds and
inclement weather cannot easily reach the shorter trees and cause damage. An additional factor that contributes to the
long life of the bristlecone pine is that this type of tree has a high percentage of resin, which prevents rot from
developing in the tree trunk and branches.

6. The best title for this passage would be…. 9. The pronoun “they” in line 2 refers to….
(A) The Size of the Bristle Cone Pine (A) Living trees on earth
(B) Three-Hundred-Year-Old Forests (B) Trees
(C) The Wonders of the Ancient World (C) The age of the trees
(D) An Amazingly Enduring Tree (D) People

7. According to the passage, approximately how 10. The pronoun “it” in line 7 refers to….
old are the oldest trees on Earth? (A) the pyramids of Egypt
(A) Two hundred years old (B) the bristlecone pine of California
(B) Three hundred years old (C) the Hanging Gardens of Babylon
(C) Five hundred years old (D) the Colossus of Rhodes
(D) Five thousand years old

8. The word "estimate" in line 2 is closest in


meaning to….
(A) measure
(B) approximate
(C) evaluate
(D) view

Center for International Language Development | 50


Questions 10-15

Venus is always considered as Earth‟s sister planet since both of them share almost similar size, mass, and
thin atmosphere. However unlike Earth, Venus has scorching surface. Data from the Pioneer spacecraft of NASA
apparently prove the theory that the high surface temperature of Venus is due to an atmospheric greenhouse effect
caused mainly by a blanket of carbon dioxide. Such a greenhouse effect is created when energy in the form of
sunlight easily passes through a planet's atmosphere, warms its surface, and is converted to heat radiation that is then
held in by the atmosphere from top to bottom.

11. The passage above mainly discusses….. 14. The word “its” in line 6 refers to ….
(A) Earth‟s atmosphere (A) Planet
(B) Earth„s Greenhouse effect (B) Sun
(C) Venus‟ hot temperature (C) Venus
(D) Venus‟s sister planet (D) Earth

12. According to the passage Venus… 15. The word “blanket” in line 5 can be replaced
(A) Is bigger than Earth by
(B) has higher temperature than Earth (A) Sheet
(C) Has thick atmosphere (B) Bed
(D) Has cooler surface than Earth. (C) Casing
(D) Cover
13. The word “them” in line 1 refers to….
(A) Similarities between Earth and Venus
(B) Atmospheres
(C) Venus
(D) Earth and Venus

Center for International Language Development | 51

Anda mungkin juga menyukai